Merge remote branch 'public/master'
[course.git] / latex / problems / Serway_and_Jewett_4 / problem11.37.tex
1 \begin{problem*}{11.37}
2 Four possible transitions for a hydrogen atom are as follows:
3 \begin{center}
4 \begin{tabular}{l l c l l}
5   (i)   & $n_i=2$; $n_f=5$ && (ii) & $n_i=5$; $n_f=3$ \\
6   (iii) & $n_i=7$; $n_f=4$ && (iv) & $n_i=4$; $n_f=7$
7 \end{tabular}
8 \end{center}
9 \Part{a} In which transition is light of the shortest wavelength emitted?
10 \Part{b} In which transition does the atom gain the most energy?
11 \Part{c} In which transition(s) does the atom lose energy?
12 \end{problem*} % problem 11.37
13
14 \begin{solution}
15 \Part{a}
16 Looking at the sized of the changes in $n$, we have
17 \begin{center}
18 \begin{tabular}{l l c l l}
19   (i)   & $\Delta n=3$ && (ii) & $\Delta n=-2$ \\
20   (iii) & $\Delta n=-3$ && (iv) & $\Delta n=3$
21 \end{tabular}
22 \end{center}
23 We know we want to release energy, so we need $\Delta n < 0$ (relaxing
24 from a more excited state).  That leaves (ii) and (iii).  We have to
25 crunch some numbers for this, since each case has a point in its
26 favor: (iii) jumps more levels, but (ii) is closer in, where the
27 levels are further apart.
28 \begin{align}
29   E_{ii} &= A \p({\frac{1}{5^2}-\frac{1}{3^2}}) = -71\E{-3}\cdot A \\
30   E_{iii} &= A \p({\frac{1}{7^2}-\frac{1}{4^2}}) = -42\E{-3}\cdot A \;,
31 \end{align}
32 so \ans{(ii)} releases the most energy.  Note that
33 $A=hcR_H=13.6\U{eV}$, but it's actual value doesn't matter because it
34 is a constant value for hydrogen.
35
36 \Part{b}
37 Now we need to compare (i) and (iv), but this is easier, since they
38 both increase by three levels, but (i) starts closer in (where the
39 levels are further apart).  Therefore, \ans{(i)} will gain the most
40 energy.
41
42 \Part{c}
43 The atom loses energy in \ans{(ii)} and \ans{(iii)} as mentioned
44 in \Part{a}.
45 \end{solution}